Does the conclusion escape you? Has understanding the tone of the passage gotten you down? Get help here.
rahul.gmat
 
 

In 1963, Congress approved the Community Mental Health

by rahul.gmat Mon Jun 04, 2007 5:08 am

In 1963, Congress approved the Community Mental Health Centers Act, which outlined plans to release the mentally ill from institutions, incorporate these individuals into their communities, and provide outpatient treatment. Leading associations of mental health professionals overwhelmingly applauded these goals and approved of these plans because, the experts said, the treatment rather than the institutional environment was the crucial element for the welfare of these patients. Within twenty years, state authorities succeeded in discharging 95% of these patients from institutional care. In 1983, however, executives from these same professional associations said that the plight of the mentally ill was worse than ever.

Which if the following, if true, best resolves the paradox in the above passage?


(A) More people were diagnosed with psychiatric disorders in 1983 than in 1963.
(B) Many mental health professionals believe that if their peers had administered the project rather than the state authorities, the results would have been better.
(C) The state budget allocation for services to the mentally ill has not increased faster than the rate of inflation.
(D) Congress agreed to fund these outpatient services, provided that the money come from cuts in other domestic programs; these cuts, however, never materialized.
(E) Many of the released patients had, at some time, been addicted to illegal narcotics.


The passage states that, on the one hand, the government was successful in releasing mental health patients from institutional care. On the other hand, according to the leading professional health associations, which explicitly supported the government's plans, the plight of the mentally ill only became worse. We need to find one statement that explains what prevented the plan from achieving its expected results.

(A) An increase in the number of people diagnosed with psychiatric disorders does not explain why the plight of psychiatric patients has gotten worse.

(B) Believing that someone else could have better implemented the plan does not explain what went wrong with the plan as it was actually implemented.

(C) The passage does not indicate or imply that adequate funding must be tied to the inflation rate. It might have been sufficient, for example, for funding increases to match the rate of inflation.

(D) CORRECT. This resolves the paradox by offering a reason why the plan failed: only half of the plan was implemented. According to the passage, the original plan supported by the leading mental health associations was "to remove the mentally ill from institutions, incorporate them into their communities, and give them outpatient treatment." While the removal was accomplished, choice D explains that the outpatient treatment services were not; patients, therefore, haven't improved.

(E) The past addictions of release patients do not explain why the plight of the patients became worse after they were released.


Here, I can clearly see that (A), (B) and (C) are out. But (E) seems so much stronger and similar to GMAT patterns (based on my experience with OG and Verbal Review). (D) dosent make that much sense, if the cuts never materialized, they cant still continue with the program for 20 years without money and then realize that its not working. Also, (D) is more about difficulties in implementation which is almost always wrong on GMAT. (E) somehow seems like the perfect GMAT answer. Medical people sais treatment is important not the environment. But seems people were addicted to drugs which they could not consume in the environment but could outside. So these people were mistaken and environment was actually playing an important role too. Could you please explain?
rahul.gmat
 
 

CR#15 from Manhattan GMAT Ques bank

by rahul.gmat Mon Jun 04, 2007 3:53 pm


Unlike juvenile diabetes, which is a genetic condition present from birth, type-2 diabetes is acquired in adulthood, generally as a result of obesity and inactivity. The number of cases of type-2 diabetes has been steadily increasing in the United States since 1970, indicating to many researchers that the American population is becoming increasingly heavy and sedentary. If the government wishes to stem the spread of the disease, it should educate the public about the dangers of an inactive, calorie-laden lifestyle and promote healthful diets and exercise.

Which of the following, if true, provides the strongest reason to believe that the proposed education program will NOT be effective?


School health programs already educate middle-school students about the issue.
The public already has access to this information through the Internet.
Food companies encourage the public to indulge in unhealthful snacks.

The government has not set aside money for such a program.
Healthful foods and exercise programs are beyond the financial means of many people.

The conclusion of the argument is that the government should educate the public about the dangers of inactivity and poor diet in order to stop the spread of type-2 diabetes. The basis for the claim is that inactivity and poor diet are the main factors in developing type-2 diabetes. We are asked to find a choice that will show that this plan likely will not work.

(A) The fact that schools educate middle school students about a disease that is generally "acquired in adulthood" does not address the effectiveness of an adult education plan sponsored by the government.

(B) The fact that the public already has access to this information through the Internet, does not say anything predictive about the effectiveness of the plan. Even with access to the information, there is a good chance that most people are not exposed to the information.

(C) Just because food companies encourage the public to indulge in unhealthful snacks, does not mean that a program that teaches them to do otherwise would not be successful.

(D) The fact that the government has not set aside money for such a program, does not say much about the projected program's effectiveness.

(E) CORRECT. Choice E states that healthful foods and exercise programs are beyond the financial means of many people. This suggests that even with the best planning, the program might not achieve its goals simply because people cannot afford to follow the program's advice.




This is a CR from the ques bank. This is exactly what I am referring to that money/implementation options are always wrong. Although here its about money in future but still, even from OG several ques hint the fact that implementation is never an issue.
JadranLee
ManhattanGMAT Staff
 
Posts: 108
Joined: Mon Aug 07, 2006 10:33 am
Location: Chicago, IL
 

by JadranLee Wed Jun 06, 2007 6:50 pm

Hi Rahul,

You're very perceptive - there is indeed a problem with CR #21 from the Question Bank. Specifically, answer choice (E) can be interpreted as the right answer, for the reasons that you so clearly stated in your message. Within the next 24 hours I am going to edit the question to remove this flaw. Thanks for bringing it to our attention!

On the other hand, answer choice (D) for CR #21 does make a reasonable right answer. It's not true that difficulties with money or with implementation of a plan are never correct answers in Critical Reasoning - see, for instance, CR #55 in the Official Guide.

For CR #21 in the Question Bank, it does make sense to say that if Congress passed a law to let mentally ill people out of institutions, but failed to pay for any outpatient care for those people, then those people might not have been helped by Congress's actions.

For CR #15 in the Question Bank, it is crucial - as you noticed - that we are talking about a hypothetical plan that the government might decide to implement IN THE FUTURE. The fact that the government has not ALREADY set aside money for that possible, FUTURE plan is irrelevant to the merits of that plan. Suppose, for instance, that your friend tells you that maybe someday she'll start her own business. The fact that she hasn't already amassed all the capital doesn't prove that her plan is flawed. She still has time to raise the money.

-Jad
gunjan_infy
 
 

by gunjan_infy Fri Dec 14, 2007 6:39 am

Could you please explain why (A) is incorrect? If there are more number of mentally ill people in 1983 than there were in 1963, then total amount of care provided is now divided in higer number of people. Hence, their plight has increased. Please guide why this explaination is wrong?

Thanks in advance.
StaceyKoprince
ManhattanGMAT Staff
 
Posts: 9350
Joined: Wed Oct 19, 2005 9:05 am
Location: Montreal
 

by StaceyKoprince Tue Dec 18, 2007 3:10 am

You made an assumption here that you can't make. How do you know the budget hasn't increased along with the number of people? You have to base your answer on the info provided (which is why correct answer D is right - it clearly addresses something that was stated in the passage).

The paradox is that the "plight of the mentally ill was worse than ever" even though most of them were no longer in institutional care (which they had thought would help the mentally ill).

This doesn't have to have anything to do with the number of people who are ill; it just means that those who are ill aren't being treated effectively even though the government seemingly implemented something that everyone thought would be very useful.
Stacey Koprince
Instructor
Director, Content & Curriculum
ManhattanPrep
Peak
 
 

by Peak Tue Feb 05, 2008 10:36 am

I understand the reasoning discussed but tht brings into question why C can't be right....For C we can think...they alloted kinda good amnt for fund but becoz of the inflation....were not able to meet the needs of patients released and tht increases the plight. For E we can think....first the habit doesn't tell nethg abt how it increased the plight...n second is that these released people were out for past 20 yrs so wht suddenly resulted in increase of plight becoz of narcotics...Pls explain further.
StaceyKoprince
ManhattanGMAT Staff
 
Posts: 9350
Joined: Wed Oct 19, 2005 9:05 am
Location: Montreal
 

by StaceyKoprince Fri Feb 15, 2008 1:00 am

Hi - please use complete sentences / spelling when posting here. We want to make sure we fully understand your enquiry!

For C, we don't know what amount of money would be adequate vs. inadequate. This choice merely says the budget did not increase faster than the rate of inflation. It could have matched the rate of inflation or been lower than the rate of inflation. Either way, the argument provides no information about why or how the rate of inflation should be tied to the budget. (You might have a little more of a case if it said that the budget increase was below the rate of inflation, but this choice leaves open the possibility that it increased at the rate of inflation, and there's no obvious reason why an increase at the rate of inflation should make things worse.)

For D, it says that the outpatient programs were not funded at all. While we don't know how much funding would be adequate, zero funding is clearly inadequate.

An earlier poster made an argument that E could be right because it could provide an alternate explanation as to why the people are worse off in the community (compared to when they were in institutions). If the mental illnesses were due to drug addictions, perhaps they were better off in the institutions bbecause they couldn't access illegal drugs. Once they were released into the community, they had access to the illegal drugs again. This is why Jad (the instructor who commented above) mentioned that we would re-work this answer choice, to avoid this problematic interpretation.
Stacey Koprince
Instructor
Director, Content & Curriculum
ManhattanPrep
Guest
 
 

by Guest Tue May 13, 2008 3:50 pm

Considering that Congress often spends money it does not have money available for, I figured even if it didn't make cuts it was still providing funding.
Guest
 
 

by Guest Thu May 15, 2008 9:20 am

Stacey/Jadran

Can one of you explain what is the money/implementation concept tested in CR? I see that the OP is saying that money/implementation issues are never the best answer in CR. Is that correct?
rfernandez
Course Students
 
Posts: 381
Joined: Fri Apr 07, 2006 8:25 am
 

by rfernandez Thu May 15, 2008 6:39 pm

In an earlier post in this thread, Jad addressed this issue, pointing to a specific question in the Official Guide that provides a counterexample. Scroll up and check it out.

Rey